control system basics

Upload: neel-patel

Post on 04-Jun-2018

224 views

Category:

Documents


0 download

TRANSCRIPT

  • 8/13/2019 Control System basics

    1/69

    1

    Lecture Notes

    Control Systems

    Xiaoping Liu

    Department of Electrical Engineering

    Lakehead University

  • 8/13/2019 Control System basics

    2/69

    2

  • 8/13/2019 Control System basics

    3/69

    Contents

    1 Mathematical Models: Differential Equations 1

    1.1 Electrical Systems: . . . . . . . . . . . . . . . . . . . . . . . . . . . . . . . . . . . . . . . . . . . . . . . . . . . 1

    1.2 Mechanical Systems . . . . . . . . . . . . . . . . . . . . . . . . . . . . . . . . . . . . . . . . . . . . . . . . . . 2

    1.2.1 Translational Motion . . . . . . . . . . . . . . . . . . . . . . . . . . . . . . . . . . . . . . . . . . . . . . 3

    1.2.2 Rotational Motion . . . . . . . . . . . . . . . . . . . . . . . . . . . . . . . . . . . . . . . . . . . . . . . 3

    1.3 Thermal Systems: Temperature Control . . . . . . . . . . . . . . . . . . . . . . . . . . . . . . . . . . . . . . . 5

    2 Laplace Transform 7

    2.1 Partial Fraction Expansion . . . . . . . . . . . . . . . . . . . . . . . . . . . . . . . . . . . . . . . . . . . . . . 8

    2.2 Using Laplace Transforms to Solve DE . . . . . . . . . . . . . . . . . . . . . . . . . . . . . . . . . . . . . . . . 9

    3 Mathematical Models: Transfer Function Representation 11

    3.1 Transfer Function for the 1st Order System . . . . . . . . . . . . . . . . . . . . . . . . . . . . . . . . . . . . . 11

    3.2 Transfer function for the 2nd order system: . . . . . . . . . . . . . . . . . . . . . . . . . . . . . . . . . . . . . 12

    3.3 Examples . . . . . . . . . . . . . . . . . . . . . . . . . . . . . . . . . . . . . . . . . . . . . . . . . . . . . . . . 12

    4 Mathematical Models: Block Diagrams 15

    5 Dynamic Responses 21

    5.1 First Order Systems . . . . . . . . . . . . . . . . . . . . . . . . . . . . . . . . . . . . . . . . . . . . . . . . . . 21

    5.1.1 Impulse Response . . . . . . . . . . . . . . . . . . . . . . . . . . . . . . . . . . . . . . . . . . . . . . . 21

    5.1.2 Step Response . . . . . . . . . . . . . . . . . . . . . . . . . . . . . . . . . . . . . . . . . . . . . . . . . 21

    5.1.3 Ramp Response . . . . . . . . . . . . . . . . . . . . . . . . . . . . . . . . . . . . . . . . . . . . . . . . 22

    5.1.4 Example . . . . . . . . . . . . . . . . . . . . . . . . . . . . . . . . . . . . . . . . . . . . . . . . . . . . . 22

    5.2 Second-Order Systems . . . . . . . . . . . . . . . . . . . . . . . . . . . . . . . . . . . . . . . . . . . . . . . . . 23

    5.2.1 Step Response . . . . . . . . . . . . . . . . . . . . . . . . . . . . . . . . . . . . . . . . . . . . . . . . . 24

    5.2.2 Performance Measures . . . . . . . . . . . . . . . . . . . . . . . . . . . . . . . . . . . . . . . . . . . . . 25

    5.2.3 Examples . . . . . . . . . . . . . . . . . . . . . . . . . . . . . . . . . . . . . . . . . . . . . . . . . . . . 25

    6 Steady-State Errors 29

    6.1 Type of System . . . . . . . . . . . . . . . . . . . . . . . . . . . . . . . . . . . . . . . . . . . . . . . . . . . . . 29

    6.2 Steady-State Errors . . . . . . . . . . . . . . . . . . . . . . . . . . . . . . . . . . . . . . . . . . . . . . . . . . 30

    6.2.1 Steady-State Errors for Closed-Loop Systems with Unity Feedback . . . . . . . . . . . . . . . . . . . . 32

    6.2.2 Steady-State Errors for General Closed-Loop Systems . . . . . . . . . . . . . . . . . . . . . . . . . . . 33

    6.2.3 Steady-State Errors for Systems with Disturbances . . . . . . . . . . . . . . . . . . . . . . . . . . . . . 35

    3

  • 8/13/2019 Control System basics

    4/69

    0 CONTENTS

    7 The Routh Stability Criterion 39

    7.1 Poles and Zeros . . . . . . . . . . . . . . . . . . . . . . . . . . . . . . . . . . . . . . . . . . . . . . . . . . . . . 39

    7.2 Stability . . . . . . . . . . . . . . . . . . . . . . . . . . . . . . . . . . . . . . . . . . . . . . . . . . . . . . . . . 39

    7.3 The Routh Stability Criterion . . . . . . . . . . . . . . . . . . . . . . . . . . . . . . . . . . . . . . . . . . . . . 40

    8 Root Locus Analysis 43

    8.1 Definition . . . . . . . . . . . . . . . . . . . . . . . . . . . . . . . . . . . . . . . . . . . . . . . . . . . . . . . . 43

    8.2 Construction . . . . . . . . . . . . . . . . . . . . . . . . . . . . . . . . . . . . . . . . . . . . . . . . . . . . . . 44

    9 PID Controllers 49

    10 Frequency Response 55

    10.1 S teady-State Response . . . . . . . . . . . . . . . . . . . . . . . . . . . . . . . . . . . . . . . . . . . . . . . . . 55

    10.2 Bode Plots . . . . . . . . . . . . . . . . . . . . . . . . . . . . . . . . . . . . . . . . . . . . . . . . . . . . . . . 56

    11 Control of Discrete Processes and PLC 61

    11.1 Introduction . . . . . . . . . . . . . . . . . . . . . . . . . . . . . . . . . . . . . . . . . . . . . . . . . . . . . . . 6111.2 PLC Programming . . . . . . . . . . . . . . . . . . . . . . . . . . . . . . . . . . . . . . . . . . . . . . . . . . . 63

    11.3 Internal Relays . . . . . . . . . . . . . . . . . . . . . . . . . . . . . . . . . . . . . . . . . . . . . . . . . . . . . 63

    11.4 Timers . . . . . . . . . . . . . . . . . . . . . . . . . . . . . . . . . . . . . . . . . . . . . . . . . . . . . . . . . . 64

    11.5 Counters . . . . . . . . . . . . . . . . . . . . . . . . . . . . . . . . . . . . . . . . . . . . . . . . . . . . . . . . . 64

  • 8/13/2019 Control System basics

    5/69

    Chapter 1

    Mathematical Models: Differential

    Equations

    1.1 Electrical Systems:

    Basic Building Blocks

    Resistor v= Ri i= vR

    Capacitor v= 1C

    idt i=Cdv

    dt

    Inductor v= L

    di

    dt i=

    1

    L

    vdt

    Example 1Consider an RC circuit shown to the left.Input: v; Output: vC.It follows from Kirchhoffs voltage law that

    v= iR + vC

    Replacingi with CdvCdt

    gives

    v= RCdvC

    dt + vC

    Moving all the terms related to the output to the left-hand side and the rest to the right-hand side, we get

    RCdvCdt

    + vC=v

    which is a first-order system.

    General form for the first order system is given by

    a1dy

    dt + a0y= b0u

    with input u and output y .

    1

  • 8/13/2019 Control System basics

    6/69

    2 CHAPTER 1. MATHEMATICAL MODELS: DIFFERENTIAL EQUATIONS

    Example 2Consider an RL circuit shown to the left.Input: v; Output: iL.It follows from Kirchhoffs voltage law that

    v= iLR+ vL

    ReplacingvL with LdiLdt

    gives

    v= RiL+ LdiLdt

    Moving all the terms related to the output to the left-hand side and the rest to the right-hand side, we get

    LdiLdt

    + RiL= v

    which is also a first-order system.

    Example 3Consider an RLC circuit shown to the left.Input: v; Output: vC.

    It follows from Kirchhoffs laws that

    i = iL+ iC

    v = iR+ vC

    Substituting the first equation to the second equation and replacing iL with 1L

    vCdt and iC with C

    dvCdt

    gives

    v= R

    1

    L

    vCdt+ C

    dvCdt

    + vC

    Differentiating the equation with respect to t produces

    dv

    dt =

    R

    LvC+ RC

    d2vC

    dt2 +

    dvC

    dt

    Moving all the terms related to the output to the left-hand side and the rest to the right-hand side, we get

    RCd2vCdt2

    +dvC

    dt +

    R

    Lvc=

    dv

    dt

    which is a second-order system.

    General form for a second-order system is given by

    a2d2y

    dt2 + a1

    dy

    dt + a0y = b1

    du

    dt + b0u

    1.2 Mechanical Systems

    The basic building blocks for mechanical systems are shown below.

  • 8/13/2019 Control System basics

    7/69

    1.2. MECHANICAL SYSTEMS 3

    1.2.1 Translational Motion

    Translational Damper

    Damper represents the type of forces experienced when we endeavor to push an object through a fluid or move an objectagainst frictional forces. The faster the object is pushed, the greater the opposing forces.

    In the ideal case, the damping or resistive force Ffis proportional to the velocity v of the piston or object, that is ,

    Ff =cv = c dxdt

    c is a damping coefficient constant. The larger the value ofc the greater the damping force at a particular velocity.

    Translational Spring:

    Suppose a spring is fixed at one end. When you try to pull or to push the spring at the other end, you are experiencingsome kinds of resistive forces exerted by the stretched or compressed spring. This force will be in the opposite direction andequal in size to the force used to stretch or compress the spring, which is proportional to the extension or compression, thatis, displacement

    Fs= kx

    where k is a spring constant called stiffness. The bigger the value of k , the greater the forces have to be to stretch orcompress the spring so the greater the stiffness.

    Translational Mass

    The mass building block exhibits the property that the bigger the mass the greater the force required to give it a specificacceleration. Newtons second law gives

    F =ma =md

    dt

    dx

    dt

    = m

    d2x

    dt2

    where the constant of proportionality between the force and the acceleration is the constant called the mass m. ais the rateof change of velocity, i.e., a = dv

    dt, and velocity v is the rate of change of displacement x, i.e., v = dx

    dt.

    1.2.2 Rotational Motion

    Rotational Damper:

    A disc is rotated in a fluid and the resistive torque fis proportional to the angular velocity (radians/second)

    f =c = cd

    dt

    is the rate of change of angular displacement (rotated angle) .

    Torsional Spring:

    The tortue produced by a rotation spring is directly proportional the angle rotated .

    s= k

    Moment of Inertia:

    Moment of inertia exhibits the property that the greater the moment of inertia Jthe greater the torque needed to produce

    an angular acceleration (radians/second2

    )

    T=J = Jdw

    dt =J

    d

    dt

    d

    dt

    = J

    d2

    dt2

    Example 4Consider a mechanical system shown to the left.Input: F; Output: x.since the net force is Fnet= Fkxcv= Fkxcdxdt . It followfrom Newtons second law that

    Fnet= ma =md2x

    dt2

  • 8/13/2019 Control System basics

    8/69

    4 CHAPTER 1. MATHEMATICAL MODELS: DIFFERENTIAL EQUATIONS

    Replacing Fnet withF kx cdxdt yields

    F kx c dxdt

    = md2x

    dt2

    Moving all the terms related to the output to the left-hand side and the rest to the right-hand side, we get

    md2x

    dt2 + c

    dx

    dt + kx = F

    Example 5Consider a mechanical system shown to the left.Input: ; Output: .Since the net torque is net = k c = k cddt . follows from Newtons second law that

    net= J =Jd2

    dt2

    Replacing net with k cddt yields

    k c ddt

    = Jd2

    dt2

    Moving all the terms related to the output to the left-hand side and the rest to the right-hand side, we get

    Jd2

    dt2 + c

    d

    dt + k =

    Example 6Armature Controlled DC MotorsIn the armature controlled dc motor, vf is kept constant, so

    if= VfRf

    . The input to the system is va and the output is . Th

    load torque is considered as a disturbance. The equivalent circufor the armature controlled dc motor is shown in the figure to thleft. The equations described the armature controlled dc motare given below.

    va = Raia+ La dia

    dt + ea (1.

    ea = Keif= K3 (1.

    d = Kifia = K4ia (1.

    f = c (1.

    d L f = Jddt

    (1.

    Substituting (1.3) and (1.4) into (1.5) gives

    K4ia L c= Jddt

    Solving this equation for ia yields

    ia = J

    K4

    d

    dt +

    c

    K4+

    1

    K4L (1.6)

    Differentiatingia produces

    diadt

    = J

    K4

    d2

    dt2 +

    c

    K4

    d

    dt +

    1

    K4

    dLdt

    (1.7)

    Substituting (1.6), (1.7) and (1.2) into (1.1), it follows that

    va = Ra

    J

    K4

    d

    dt +

    c

    K4+

    1

    K4L

    + La

    J

    K4

    d2

    dt2 +

    c

    K4

    d

    dt +

    1

    K4

    dLdt

    + K3

  • 8/13/2019 Control System basics

    9/69

  • 8/13/2019 Control System basics

    10/69

    6 CHAPTER 1. MATHEMATICAL MODELS: DIFFERENTIAL EQUATIONS

    Example 7 Consider a thermal system shown to the left. The ambienttemperature isTaand the temperature inside the box is T. The heat producedby the heater flows into the box at a rate qi. The thermal capacitance of theair inside the box is Cand the thermal resistance of the box wall is R. Findthe model for the system with input qi and output T.Solution: The heat flow rate out of the box through the box wall is

    qo =

    T

    Ta

    R

    The net heat flow rate into the box is qi qo. The heat required to raise theinside temperature fromT to T+ dT is C dT, which implies that the heat raterequired is CdT

    dt. As a result,

    CdT

    dt = qi q0 = qi T Ta

    R

    Rearranging the equation gives

    CdT

    dt +

    T

    R = qi+

    TaR

    whereTa can be considered a disturbance to the system.

    Example 8Consider a thermometer at temperatureTwhich has just been insertedinto a liquid at temperature TL.If the thermal resistance to heat flow from the liquidto the thermometer is R, then the heat flow rate from the liquid to the thermometeris

    q=TL T

    R

    which is used up to raise the thermometer temperature from T to T+ dT. Then wehave

    q=TL T

    R =C

    dT

    dt

    that is,TL T

    R =C

    dT

    dt

    Rearranging the equation gives

    CdT

    dt +

    T

    R=

    TLR

    which is a first-order system with input TL and output T .

  • 8/13/2019 Control System basics

    11/69

    Chapter 2

    Laplace Transform

    The Laplace transform is a method of transforming differential equations into more easily handled algebraic equations.. Letf(t) be a function defined on [0, T]. Then, its Laplace transform is defined by

    L[f(t)] =F(s) =

    0

    f(t)estdt

    The inverse Laplace transform is defined by

    f(t) = L1[F(s)] = 12j

    b+jbj

    F(s)estds

    The lower case letters are used to denote the functions in time domain. The upper case letters are used to denote thefunctions in s domain.

    As an example, let us calculate the Laplace transform for a unit step function

    1(t) =

    1 t 00 t < 0

    Its Laplace transform is

    L[1(t)] =0

    estdt=

    1

    sest

    0

    =1

    s

    Generally speaking, its not easy to calculate the Laplace transform and the inverse Laplace transform for a given function.Fortunately, for those functions which are commonly used in this course, we dont need to calculate them ourself. There isLaplace Transform Table available, where you can find what you want. But it is useful to remember some commonly usedLaplace transforms, for instance:

    L[(t)] = 1, L[t] = 1s2

    , L[eat] = 1s + 1

    , L[sin t] = s2 + 2

    , L[cos t] = ss2 + 2

    Basic Rules:

    L[f

    1(t) + f

    2(t)] =F

    1(s) + F

    2(s)

    L1[F

    1(s) + F

    2(s)] =f

    1(t) + f

    2(t)

    L[f1(t) f2(t)] =F1(s) F2(s) L1[F1(s) F2(s)] =f1(t) f2(t)L[af(t)] =aF(s) L1[aF(s)] =af(t)L[f(t T)] = eTsF(s) L1[eTsF(s)] =f(t T)L[ d

    dtf(t)] =sF(s) f(0)

    L[ d2dt2

    f(t)] =s2F(s) sf(0) df(0)dt

    L[ dndtn

    f(t)] =snF(s) sn1f(0) dn1f(0)dtn1

    L[t0

    f(t)dt] = 1s

    F(s)Limitt0

    f(t) =Limits

    sF(s) The initial value theorem

    Limitt

    f(t) =Limits0

    sF(s) The final value theorem

    7

  • 8/13/2019 Control System basics

    12/69

  • 8/13/2019 Control System basics

    13/69

    2.2. USING LAPLACE TRANSFORMS TO SOLVE DE 9

    2.2 Using Laplace Transforms to Solve DE

    1. Determine initial conditions

    2. Take Laplace transform to change the differential equations to algebraic equations

    3. Solve the algebraic equations for unknown variables

    4. Carry out partial fraction expansion to put the resulting equations into the sum of fractions which can be found in

    the Laplace transform table.5. Take inverse Laplace transform to change s-domain functions into t-domain functions

    Example 3Solve the differential equation

    RCdvCdt

    +vC=v

    with zero initial condition and step input v =V 1(t), which is a model forthe RC circuit to the left.Solution: Taking Laplace transform on both sides gives

    RCsVC(s) + VC(s) =V

    s

    Solving forVC(s) yields

    VC(s) = V

    s(RCs + 1)

    Converting it into the form that you can find in Laplace transform table, it follows that

    VC(s) =V1

    RC

    s

    s + 1RC

    Taking the inverse transform produces

    VC(t) =V

    1 e tRC

    Example 4Solve the differential equation

    CdTdt

    + 1R

    T= 1R

    TL, T(0) = 0

    which is the mathematical model for the thermal system shown in the figure to theleft.Solution: TakingLtransform gives

    CsT(s) + 1

    RT(s) =

    1

    RTL(s)

    Solving forT(s) produces

    T(s) =1R

    Cs + 1R

    TL(s) = 1

    RCs + 1TL(s)

    Case 1 Unit-impulse responseTL(t) =(t) = TL(s) = 1. The temperature in s-domain is given byT(s) =

    1

    s+ 1=

    1

    s + 1

    with =RC. TakingL1 transform yieldsT(t) =

    1

    e

    t

    Case 2 Unit-step responseTL(t) = 1(t) = TL(s) = 1s . The temperature in s-domain is given by

    T(s) = 1

    s+ 1

    1

    s =

    1

    s(s + 1

    )

  • 8/13/2019 Control System basics

    14/69

    10 CHAPTER 2. LAPLACE TRANSFORM

    TakingL1 transform givesT(t) = 1 e t

    Case 3Unit ramp response TL(t) =t = TL(s) = 1s2 . The temperature in s-domain is given by

    T(s) = 1

    s+ 1

    1

    s2 =

    1

    s(s + 1

    )

    TakingL1 transform producesT(t) = t (1 e

    t )

    1

  • 8/13/2019 Control System basics

    15/69

    Chapter 3

    Mathematical Models: Transfer Function

    Representation

    Suppose we have a system where the inputs u is related to the output y by the differential equation:

    a2d2ydt2

    + a1dydt

    + a0y = b1dudt

    + b0u

    wherea2, a1, a0, b1,and b0 are constants. If all the initial conditions are zero, then the Laplace transform of this equation is

    a2s2Y(s) + a1sY(s) + a0Y(s) =b1sU(s) + b0U(s)

    Solving forY(s) gives

    Y(s) = b1sU(s) + b0U(s)

    a2s2 + a1s +a0=

    b1s+ b0a2s2 + a1s + a0

    U(s)

    Hence,

    Y(s)

    U(s) = b

    1s+ b

    0a2s2 + a1s + a0

    This is the transfer function of the system. The transfer functionG(s) of a linear system is defined as the ratio of the Laplacetransform of the output variable Y(s) to the Laplace transform of the input variableU(s),with all initial conditions assumedto be zero.

    G(s) =Y(s)

    U(s)

    For the system described by the differential equation above

    G(s) =Y(s)

    U(s)=

    b1s + b0a2s2 + a1s+ a0

    3.1 Transfer Function for the 1st Order System

    General Form

    Differential Equation a1dydt

    + a0y= b0u zero initial conditionsLaplace Tansform a1sY(s) + a0Y(s) =b0U(s)

    Transfer Function G(s) = Y(s)U(s) =

    b0a1s+a0

    Standard Form

    Differential Equation dydt

    + y= Ku zero initial conditionsLaplace Tansform sY(s) + Y(s) =KU(s)

    Transfer Function G(s) = Y(s)U(s) =

    Ks+1

    11

  • 8/13/2019 Control System basics

    16/69

    12 CHAPTER 3. MATHEMATICAL MODELS: TRANSFER FUNCTION REPRESENTATION

    3.2 Transfer function for the 2nd order system:

    General Form

    Differential Equation a2d2ydt2

    + a1dydt

    + a0y= b0u zero initial conditionsLaplace Tansform a2s2Y(s) +a1sY(s) + a0Y(s) =b0U(s)

    Transfer Function G(s) = Y(s)U(s) =

    b0a2s2+a1s+a0

    Standard Form

    Differential Equation d2y

    dt2 + 2n

    dydt

    + 2ny= K2nu zero initial conditions

    Laplace Tansform s2

    Y(s) + 2nsY(s) + 2

    nY(s) =K 2

    nU(s)Transfer Function G(s) =

    Y(s)U(s) =

    K2ns2+2ns+2n

    3.3 Examples

    Example 1Find the transfer function for the differential equation

    RCdvCdt

    + vC=v

    which is a model for the RC circuit to the left.Solution: Taking Laplace transform on both sides, with zero initial conditions,gives

    RCsVC(s) + VC(s) = V(s)

    Solving forVC(s) yields

    VC(s) = V(s)

    RCs + 1

    Then the transfer function is given by

    G(s) =VC(s)

    V(s) =

    1

    RCs + 1

    Example 2Find the transfer function for the differential equation

    L

    R

    diLdt

    + iL= v

    which is a model for the RL circuit to the left.Solution: Taking Laplace transform on both sides, with zero initial conditions,gives

    L

    RsIL(s) + IL(s) =V(s)

    Solving forIL(s) produces

    IL(s) = 1LR

    s+ 1V(s)

    which implies that

    G(s) = IL(s)

    V(s) =

    1LR

    s+ 1

    Example 3Find the transfer function for the differential equation

    RCd2vCdt2

    +dvC

    dt +

    R

    LvC=

    dv

    dt

    which is a model for the RLC circuit to the left.Solution: Taking Laplace transform on both sides, with zero initialconditions, gives

    RCs2VC(s) + sVC(s) +R

    LVC(s) = sV(s)

  • 8/13/2019 Control System basics

    17/69

    3.3. EXAMPLES 13

    Solving forVC(s) produces

    VC(s) = s

    RCs2 + s+ RL

    V(s)

    which implies that

    G(s) =IL(s)

    V(s) =

    s

    RCs2 + s + RL

    Example 4 Find the transfer function for the differential equatio

    LaJ

    K4

    d2

    dt2 +

    RaJ+ Lac

    K4

    d

    dt +

    Rac + K3K4K4

    = va RaK4

    L LK

    which is a model for the armature controlled dc motor shown tthe left.Solution: Taking Laplace transform on both sides, with zeinitial conditions, gives

    LaJ

    K4

    s2(s) +RaJ+ Lac

    K4

    s(s) +Rac + K3K4

    K4

    (s)

    = Va(s) RaK4

    L(s) s LaK4

    L(s)

    Solving this equation for (s) produces

    (s) = K4

    LaJs2 + (RaJ+ Lac)s + Rac+ K3K4Va(s) Las + Ra

    LaJs2 + (RaJ+ Lac)s + Rac+ K3K4L(s)

    = K4

    (Las + Ra)(Js+ c) + K3K4Va(s) Las + Ra

    (Las + Ra)(Js+ c) + K3K4L(s)

    Example 5Find the transfer function for the differential equation

    CdT

    dt +

    1

    RT=

    1

    RTL

    which is the mathematical model for the thermal system shown in the figure to theleft.Solution: TakingLtransform gives

    CsT(s) + 1

    RT(s) =

    1

    RTL(s)

    Solving forT(s) produces

    T(s) =1R

    Cs + 1R

    TL(s) = 1

    RCs + 1TL(s)

    Then, the transfer function is given by

    G(s) = T(s)TL(s)= 1RCs + 1

  • 8/13/2019 Control System basics

    18/69

    14 CHAPTER 3. MATHEMATICAL MODELS: TRANSFER FUNCTION REPRESENTATION

  • 8/13/2019 Control System basics

    19/69

    Chapter 4

    Mathematical Models: Block Diagrams

    Consider a transfer functionG(s) = Y(s)U(s) withU(s) andY(s) being its input and output. It is obvious thatY(s) =G(s)U(s),

    which can be represented by a block diagram as shown in the figure below.

    Arrows are used to represent the directions of signal flow.

    Comparator can be represented by a summing point.

    Where a signal is taken off from some point in a signal path, the take-off point is used.

    Closed-loop system

    Forward path is used for those elements through whichsignal passes when moving in the direction from input

    output for a system as a whole, as shown in the figure tthe left. The transfer function of the forward path is callethe feedforward transfer function.Feedback path is used for those elements through whichsignal passes when being fed back from the output towarthe input, as shown in the figure to the left. The transffunction of the feedback path is called the feedback transffunction.

    Series Connection: The transfer function of a system composed of several subsystems connected in series is the productof transfer functions of individual subsystems, as shown in the figure below.

    Parallel Connection: The transfer function of the system composed of two systems connected in parallel, as shown inthe figure below, is the sum of transfer functions of individual subsystems.

    15

  • 8/13/2019 Control System basics

    20/69

    16 CHAPTER 4. MATHEMATICAL MODELS: BLOCK DIAGRAMS

    Feedback Connection: The open-loop transfer function is defined as the ratio of the feedback signal Uf(s) to theactuating error signalU(s) Uf(s), which is given by

    Go(s) = Uf(s)

    U(s) Uf(s) =G(s)H(s)

    The closed-loop transfer function is defined as the ratio of the output signal Y(s) to the input signal X(s), which is derivedas follows:

    Y(s) = G(s)X(s)

    = G(s)[U(s) Uf(s)]= G(s)[U(s) H(s)Y(s)]= G(s)U(s) G(s)H(s)Y(s)

    which implies that

    [1 + G(s)H(s)]Y(s) =G(s)U(s)

    Therefore, the transfer function is

    Gc(s) = G(s)

    1 + G(s)H(s)

    Example 1: Find transfer functions for the systems shown in thfigure to the left.Solution: The transfer function for the first system is

    G(s) = G1(s) + G2(s) = 1

    s + 1+ 5 =

    1

    s+ 1+

    5(s+ 1)

    s + 1 =

    5s +

    s +

    The transfer function for the second system is

    Gc(s) = G(s)

    1 G(s)H(s) =2

    s+1

    1 2s+1 5s

    =2

    s+1s+110s

    s+1

    = 2

    9s + 1Example 2 Armature controlled dc motor

  • 8/13/2019 Control System basics

    21/69

    17

    The equivalent circuit for the armature controlled dc motor shown in the figure to the left. The system can be divided infour parts, armature winding,Ea versus,d versusia, and loaThe transfer functions for all parts are derived below.Armature Winding: It follows from Kirchhoffs voltage la

    thatva ea= Raia+ La dia

    dt

    Taking Laplace transform on both sides gives

    Va(s) Ea(s) =RaIa(s) + LasIa(s)

    that is,

    Ia(s)

    Va(s) Ea(s) = 1

    Las+ Ra

    Ea versus : It is obvious thatea= K3

    Taking Laplace transform on both sides givesEa(s) = K3(s)

    that is,Ea(s)

    (s) =K3

    d versus ia: It is easily seen thatd = K4ia

    Taking Laplace transform on both sides givesd(s) = K4Ia(s)

    that is,

    d(s)Ia(s)

    =K4

    Load: It follows from Newtons second law that

    d L f =Jddt

    Replacingf byc , we get

    d L c= Jddt

    which is equivalent to

    d L = Jddt

    + c

    Taking Laplace transform on both sides gives

  • 8/13/2019 Control System basics

    22/69

    18 CHAPTER 4. MATHEMATICAL MODELS: BLOCK DIAGRAMS

    d(s) L(s) = J s(s) + c(s)that is,

    Ia(s)

    d(s) L(s) = 1

    Js + c

    These blocks are shown in the figure in previous page.

    Now we can put all four block diagrams together to create a block diagram for the armature controlled dc motor, whichis shown in the figure below.

    In the following, we will determine the transfer function Ga(s) from Va(s) to (s) and the transfer function G(s) fromL(s) to (s). Ga(s) can be determined by letting L(s) = 0. The process for determining Ga(s) is shown in the figurebelow, from which we get

    Ga(s) = K4

    (Las + Ra)(Js+ c) + K3K4

  • 8/13/2019 Control System basics

    23/69

    19

    G(s) can be determined by letting Va(s) = 0. The process for determining Ga(s) is shown in the figure below, fromwhich we get

    G(s) = Las+ Ra

    (Las+ Ra)(Js+ c) + K3K4

    The output from the armature controlled dc motor is given by

    (s) = Ga(s)Va(s) + G(s)[L(s)] = K4(Las+ Ra)(Js+ c) +K3K4

    Va(s) Las+ Ra(Las+ Ra)(Js+ c) + K3K4

    L(s)

  • 8/13/2019 Control System basics

    24/69

    20 CHAPTER 4. MATHEMATICAL MODELS: BLOCK DIAGRAMS

  • 8/13/2019 Control System basics

    25/69

    Chapter 5

    Dynamic Responses

    There are four kinds of responses, namely impulse response, step response, ramp response, and frequency response, whichare commonly used in control systems. The impulse response is the output of the system corresponding to an impulse input.The step response is the output of the system corresponding to a step input. The ramp response is the output of the systemcorresponding to a ramp input. The frequency response is the output of the system corresponding to a sinusoidal input.

    5.1 First Order Systems

    A first-order system can be described by a differential equation or a transfer function as shown below.

    = a1a0

    , K= b0a0

    a1dydt

    + a0y= b0u dydt + y = KuL1 L L1 LG(s) = b0

    a1s+a0 G(s) = K

    s+1

    a1 = , a0 = 1, b0 = K

    5.1.1 Impulse Response

    Inputu = A(t) or U(s) =AOutput in s-domain is given by

    Y(s) =G(s)U(s) = KA

    s+ 1

    Output in t-domain is given by (taking inverse Laplace transform)

    y(t) =KA

    e

    t

    5.1.2 Step Response

    Inputu = A 1(t) or U(s) = AsOutput in s-domain is given by

    Y(s) =G(s)U(s) = KA

    s( s+ 1)=K A

    1

    s(s + 1

    )

    Output in t-domain is given by (taking inverse Laplace transform)

    y(t) = KA(1 e t ) = KA KAe t

    where the first term is called the forced response or steady-state response, which is caused by the input, whereas the secondterm is referred to as transient response or dynamic response, which represents the transient process experienced by thesystem after an input is applied.

    21

  • 8/13/2019 Control System basics

    26/69

    22 CHAPTER 5. DYNAMIC RESPONSES

    Kis called a steady-state gain, which is the ratio between the steady-state output and input. is referred to as a timeconstant, which shows how fast the system responds to the step input. The output will reach to 60% of its steady-state valueat onemoment, 95% at 3 times .

    5.1.3 Ramp Response

    Inputu = At or U(s) =

    A

    s2

    Output in s-domain is given by

    Y(s) =G(s)U(s) = AK

    s2( s+ 1)=K A

    1

    s2(s + 1

    )

    Output in t-domain is given by (taking inverse Laplace transform)

    y(t) =K A

    t

    1 e t

    5.1.4 Example

    Example: A thermocouple has the dynamics of

    10dTodt

    + To= 30 106Ti

    with output To in volt and input Ti inC. Answer the following questions:

    1. Transfer function.

    2. Time constant and steady-state gain.

    3. The final steady-state value when there is a step input of 100C.

    4. What will be the output of the thermocouple 5s after it was subject to a temperature impulse of 100C by suddenlyand very briefly coming into contact with a hot subject of 100C.

    5. When the thermocouple is subject to a steadily rising temperature input of 5C/s, what will be the thermocoupleoutput after 12s.

    Solution:

    1. Taking Laplace transform on both sides of the differential equation gives

    10sTo(s) + To(s) = 30 106Ti(s)

    Solving this equation for To(s) produces

    To(s) =30 106

    10s + 1 Ti(s)

    which implies that the transfer function is

    G(s) =30 106

    10s + 1

    2. Since G(s) is in the standard form, = 10sand K= 30 106V /C.

    3. The input is Ti(t) = 100 1(t) and its Laplace transform is Ti(s) = 100s . It follows from the final value theorem that

    To() = lims0

    sTo(s) = lims0

    sG(s)Ti(s) = lims0

    s30 106

    10s + 1

    100

    s = lim

    s03000 106

    10s + 1 = 3000 106V

  • 8/13/2019 Control System basics

    27/69

    5.2. SECOND-ORDER SYSTEMS 23

    4. The input is Ti(t) = 100(t) and its Laplace transform is Ti(s) = 100. Then, the output in s-domain is

    To(s) =G(s)Ti(s) =30 106

    10s + 1 100 =3000 10

    6

    10s + 1 = 3000 106

    110

    s + 110

    Taking inverse Laplace transform gives

    To(t) =

    3000

    106

    10 e t10

    ThereforeTo(t) at t=5s is given by

    To(5) = 3000 106

    10 e

    5

    10 = 1.8 104V

    5. The input is Ti(t) = 5tand its Laplace transform is Ti(s) = 5s2

    . Then, the output in s-domain is

    To(s) =G(s)Ti(s) =30 106

    10s + 1

    5

    s2 =

    150 106s2(10s + 1)

    = 150 106110

    s2(s + 110 )

    Taking inverse Laplace transform gives

    To(t) = 150 106

    t 10

    1 e t10

    ThereforeTo(t) at t=12s is given by

    To(12) = 150 106

    12 10

    1 e1210

    = 7.5 104V

    5.2 Second-Order Systems

    A second-order system can be described by a differential equation or a transfer function as shown below.

    2n = a1a2

    , 2n= a0a2

    , K2n = b0a2

    a2d2ydt2

    + a1dydt

    +a0y= b0u d2y

    dt2 + 2n

    dydt

    + 2ny= K2nu

    L1 L L1 LG(s) = b0

    a2s2+a1s+a0 G(s) = K2n

    s2+2ns+2na2= 1, a1 = 2n, a0 =

    2n, b0 = K

    2n

    The equation obtained by letting the denominator of a transfer function equal to zero is called the characteristic equation.For the second order system, the characteristic equation is as follows:

    s2 + 2ns + 2n = 0

    Roots of this characteristic equation are

    s1,2 =2n

    (2n)2 42n2

    (5.1)

    =2n

    422n 42n2

    (5.2)

    =n n

    2 1 (5.3)

    Case 1: >1. The system is called overdamped. In this case, the characteristic equation has two different real roots.Case 2: = 1. the system is called critically damped. In this case, the characteristic equation has two equal real roots.Case 3:

  • 8/13/2019 Control System basics

    28/69

  • 8/13/2019 Control System basics

    29/69

    5.2. SECOND-ORDER SYSTEMS 25

    Now let a = s1= n. Then,y(t) =K

    1 ent ntent

    Case 3:

  • 8/13/2019 Control System basics

    30/69

    26 CHAPTER 5. DYNAMIC RESPONSES

    The percent overshoot is

    %Overshoot = e

    12 100% =e 0.4410.442 100% = 21%

    The 5% settling time is

    ts = 3

    n=

    3

    0.44 2 = 3.4s

    Example 2An armature controlled dc motor speed system is described by the following differential equation:

    d2

    dt2 + 5

    d

    dt+ 100= 100va

    What is the 5% settling time for the system?

    Solution: Comparing the model above with the standard form

    d2

    dt2 + 2n

    d

    dt + 2n= K

    2nva

    produces

    2n= 100 n = 102n = 5 = 5

    2n= 0.25

    K2n = 100 K=100

    2n= 1

    Thus, the 5% settling time is

    ts= 3

    n=

    3

    0.25 10= 1.2s

    Example 3A second-order system has an overshoot of 10% and a rise time of 0.4s when subject to a step input. Findthe damping ratio, damped frequency and natural frequency.

    Solution: It follows from the definition of the percent overshoot that

    e

    12 100% = 10%

    that is

    e

    12 = 0.1 1 2 = ln 0

    .1 = 2.3

    Multiplying both sides by

    1 2 gives

    = 2.3

    1 2 22 = 2.32(1 2)

    Solving foryields

    = 2.32 + 2.32

    = 0.6

    Since the rise time is given by tr = 2d

    , it is obtained that

    d=

    2tr=

    2 0.4= 3.9rad/s

    It follows from d= n

    1 2 thatn =

    d1 2 =

    3.91 0.62 = 4.9rad/s

    Example 4A second-order system has a damping ratio of 0.4, a natural frequency of 10rad/s and a steady-state gain of10. Find the transfer function, percent overshoot, rise time, and unit step response.

  • 8/13/2019 Control System basics

    31/69

  • 8/13/2019 Control System basics

    32/69

    28 CHAPTER 5. DYNAMIC RESPONSES

    Example 6A robot arm has a transfer function of

    G(s) = 9

    (s+ 3)2

    Find the unit ramp response.

    Solution: The input is u(t) = t and its Laplace transform is U(s) = 9s2

    . Hence, the output in s-domain is

    Y(s) =G(s)U(s) = 9

    s2(s+ 3)2

    Carrying out partial fraction expansion, we have

    9

    s2(s+ 3)2 =

    A

    s +

    B

    s2+

    C

    s+ 3+

    D

    (s + 3)2

    Multiplying both sides by s2(s + 3)2 gives

    9 =As(s + 3)2 + B(s+ 3)2 + Cs2(s + 3) + Ds2

    Let s = 3. Then, 9 = 9D, that is D = 1.Let s = 0. Then, 9 = 9B, that is, B = 1.Let s = 1 and s = 1, respectively. Then, we get

    9 = 16A + 16B+ 4C+ D

    9 =4A + 4B+ 2C+ D

    Replacing B and D by 1 produces

    8 = 16A + 4C 2 = 4A + C4 =4A + 2C 2 = 2A + C

    Subtracting the second equation from the first equation gives 4 = 6A, that isA = 23 . It follows from the second equationthat C= 2 + 2A= 2 43 = 23 . As a result, we have

    Y(s) = 2

    3

    s +

    1

    s2+

    23

    s+ 3+

    1

    (s + 3)2

    By taking inverse Laplace transform, the output in t-domain is given by

    y(t) = 23

    1(t) + t+2

    3e3t + te3t

  • 8/13/2019 Control System basics

    33/69

    Chapter 6

    Steady-State Errors

    6.1 Type of System

    Consider a closed-loop system with unity feedback described in the figure below.

    Note that the open-loop transfer function of the closed-loop system is Go(s), which is the same as the feedforward transferfunction. Generally speaking, annth order system can be put into the form of

    Go(s) = K(sm + bm1sm1 + + b1s+ b0)

    sq(snq + anq1snq1 + + a1s + a0)The integer qis called the type or class of the system. The closed-loop transfer function is given by

    Gc(s) = Go(s)

    1 + Go(s)

    ExampleIdentify the types of the closed-loop systems with unity feedback, which have the following open-loop transferfunctions:

    5s+2 Type 02(s+1)

    s2+2s+1 Type 06

    s(s+3) Type 12

    s2+4s = 2s(s+4) Type 1

    2(s+3)s2(s2+2s+1) Type 2

    29

  • 8/13/2019 Control System basics

    34/69

    30 CHAPTER 6. STEADY-STATE ERRORS

    The types of general closed-loop systems, as shown in the figure below, can be identified by the following procedure.

    1. Convert the system to one with unity feedback as shown in the figure above.

    2. Calculate the open-loop transfer function for the equivalent closed-loop system with unity feedback.

    3. Convert the open-loop transfer function into the form as in the definition of the type of a system

    ExampleSee the figures on the next page.

    6.2 Steady-State Errors

    The following constants are useful for the calculation of steady-state errors.The proportional error constant kp is defined as

    kp = lims0

    Go(s)

    The velocity error constant kv is defined as

    kv = lims0 sGo(s)

    The acceleration error constant ka is defined as

    ka = lims0

    s2Go(s)

    It is not difficult to verify the table below.

    Type 0 Type 1 Type 2

    kp Kb0a0

    kv 0 K

    b0a0

    ka 0 0 K

    b0a0

  • 8/13/2019 Control System basics

    35/69

    6.2. STEADY-STATE ERRORS 31

  • 8/13/2019 Control System basics

    36/69

    32 CHAPTER 6. STEADY-STATE ERRORS

    Steady-state error is defined as

    ess(t) = limt

    e(t) = lims

    0sE(s)

    wheree(t) =u(t) y(t) and E(s) =U(s) Y(s) with input u(t) or U(s) and output y(t) or Y(s).

    6.2.1 Steady-State Errors for Closed-Loop Systems with Unity Feedback

    In the closed-loop system with unity feedback as shown in the figure below, the error in s-domain is given by

    E(s) =U(s) Y(s) =U(s) Go(s)E(s)

    which implies that

    E(s) = 1

    1 + Go(s)U(s)

  • 8/13/2019 Control System basics

    37/69

  • 8/13/2019 Control System basics

    38/69

    34 CHAPTER 6. STEADY-STATE ERRORS

    ExampleSee the figure below.

    Go(s) = 6(s + 3)

    s3 + 5s2 12kp = lim

    s0Go(s) = lim

    s06(s + 3)

    s3 + 5s2 12= 18

    12= 3

    2

    kv = lims0

    sGo(s) = lims0

    6s(s + 3)

    s3 + 5s2 12= 0

    12= 0

    ka = lims0

    s2Go(s) = lims0

    6s2(s+ 3)

    s3 + 5s2 12= 0

    12= 0

    ess =

    11+kp

    = 11 3

    2

    = 2 U(s) = 1s

    1kv

    = U(s) = 1s2

    1ka = U(s) = 1s3

  • 8/13/2019 Control System basics

    39/69

    6.2. STEADY-STATE ERRORS 35

    It is important to note that steady-state errors can also be calculated directly, as shown below.

    Gc(s) = Y(s)

    U(s) =

    6s(s+2)

    1 + 6s(s+2)

    1s+3

    = 6(s + 3)

    s(s + 2)(s+ 3) + 6=

    6s + 18

    s3 + 5s2 + 6s + 6

    E(s) = U(s) Y(s) =U(s) Gc(s)U(s) = [1 Gc(s)]U(s) = 1 6s + 18s3 + 5s2 + 6s + 6U(s)=

    s3 + 5s2 + 6s + 6 6s 18s3 + 5s2 + 6s + 6

    U(s) = s3 + 5s2 12s3 + 5s2 + 6s + 6

    U(s)

    ess = lims0

    sE(s)

    =

    lims0 s s3+5s212

    s(s3+5s2+6s+6) U(s) = 1

    s

    lims0 s s3+5s212

    s2(s3+5s2+6s+6) U(s) = 1s2

    lims0 s s3+5s212

    s3(s3+5s2+6s+6) U(s) = 1

    s3

    =

    126 = 2 U(s) = 1s120 = U(s) = 1s2120 =

    U(s) = 1

    s3

    6.2.3 Steady-State Errors for Systems with Disturbances

    The direct method is used to calculate steady-state errors for systems with disturbances as shown in the figure below.

    To this end, the output in s-domain is computed first.

    Y(s) = G2(s)[Ud(s) + X1(s)] =G2(s)[Ud(s) + G1(s)X2(s)]

    = G2(s){Ud(s) + G1(s)[Ui(s) X3(s)]} =G2(s){Ud(s) + G1(s)[Ui(s) H(s)Y(s)]}= G2(s)Ud(s) + G2(s)G1(s)Ui(s) G2(s)G1(s)H(s)Y(s)

    from which one gets

    Y(s) = G2(s)

    1 + G2(s)G1(s)H(s)Ud(s) +

    G2(s)G1(s)

    1 + G2(s)G1(s)H(s)Ui(s)

    The error in s-domain is given by

    E(s) = Ui(s) Y(s) =Ui(s)

    G2(s)

    1 + G2(s)G1(s)H(s)Ud(s) +

    G2(s)G1(s)

    1 + G2(s)G1(s)H(s)Ui(s)

    = 1 + G2(s)G1(s)H(s) G2(s)G1(s)

    1 + G2(s)G1(s)H(s) Ui(s) G2(s)

    1 + G2(s)G1(s)H(s)Ud(s)

    The steady-state error is ess= lims0 sE(s).

  • 8/13/2019 Control System basics

    40/69

    36 CHAPTER 6. STEADY-STATE ERRORS

    ExampleIn the armature controlled dc motor as shown in the figure below, Va(s) = 1s

    and L(s) = 5s

    . Find ess|L=0,ess|Va=0, ess.

    Solution: First let G1(s) = 500.1s+1 , G2(s) =

    1s+1 , and H(s) = 0.2. Then we have

    1 + G2(s)G1(s)H(s) = 1 + 1

    s + 1

    50

    0.1s + 1 0.2

    = (0.1s + 1)(s+ 1) + 10

    (0.1s + 1)(s + 1) =

    0.1s2 + 1.1s + 11

    (0.1s + 1)(s + 1)

    1 + G2(s)G1(s)H(s) G2(s)G1(s) = 0.1s2 + 1.1s + 11

    (0.1s + 1)(s+ 1) 1

    s+ 1

    50

    0.1s + 1

    = 0.1s2 + 1.1s 39

    (0.1s + 1)(s+ 1)

    1 + G2(s)G1(s)H(s) G2(s)G1(s)1 + G2(s)G1(s)H(s)

    =

    0.1s2+1.1s39(0.1s+1)(s+1)

    0.1s2+1.1s+11(0.1s+1)(s+1)

    = 0.1s2 + 1.1s 390.1s2 + 1.1s + 11

    G2(s)

    1 + G2(s)G1(s)H(s) =

    1s+1

    0.1s2+1.1s+11(0.1s+1)(s+1)

    = 0.1s + 1

    0.1s2 + 1.1s + 11

    E(s) = 1 + G2(s)G1(s)H(s) G2(s)G1(s)

    1 + G2(s)G1(s)H(s) Va(s) G2(s)

    1 + G2(s)G1(s)H(s)[L(s)]

    = 1 + G2(s)G1(s)H(s) G2(s)G1(s)

    1 + G2(s)G1(s)H(s) Va(s) +

    G2(s)

    1 + G2(s)G1(s)H(s)L(s)

    E(s) |L=0 = 1 + G2(s)G1(s)H(s) G2(s)G1(s)

    1 + G2(s)G1(s)H(s) Va(s)

    = 0.1s2 + 1.1s 39s(0.1s2 + 1.1s + 11)

    ess|L=0 = lims0 sE(s) |L=0= lims00.1s2 + 1.1s

    39

    0.1s2 + 1.1s + 11= 39

    11

    E(s) |Va=0 = G2(s)

    1 + G2(s)G1(s)H(s)L(s)

    = 5(0.1s + 1)

    s(0.1s2 + 1.1s + 11)

    ess|Va=0 = lims0

    sE(s) |Va=0= lims0

    5(0.1s + 1)

    0.1s2 + 1.1s + 11=

    5

    11

    E(s) = 1 + G2(s)G1(s)H(s) G2(s)G1(s)

    1 + G2(s)G1(s)H(s) Va(s) +

    G2(s)

    1 + G2(s)G1(s)H(s)L(s)

    = E(s) |L=0 +E(s) |Va=0

  • 8/13/2019 Control System basics

    41/69

    6.2. STEADY-STATE ERRORS 37

    Therefore, we have

    ess= lims0

    sE(s) = lims0

    sE(s) |L=0 + lims0

    sE(s) |Va=0=ess|L=0 +ess|Va=0= 39

    11+

    5

    11=

    3411

  • 8/13/2019 Control System basics

    42/69

    38 CHAPTER 6. STEADY-STATE ERRORS

  • 8/13/2019 Control System basics

    43/69

    Chapter 7

    The Routh Stability Criterion

    7.1 Poles and Zeros

    In general, the transfer function G(s) of a system can be represented by

    G(s) = K(sm + bm1sm1 + + b1s + b0)

    sn + bn1sn1 + + a1s + a0whereKis the gain of the system.

    Now assume that z1, z2, , zm are roots of the numerator, and p1, p2, , pn are roots of the denominator. Then, z1,z2, ,zm are called zeros of the system, and p1, p2, , pn are referred to as poles of the system. In addition, the transferfunctionG(s) can be written as

    G(s) =K(s z1)(s z2) (s zm)

    (s p1)(s p2) (s pn)We have the following observations:

    1. The zeros are the values of s for which the transfer function becomes zero and the poles are the values of s for which

    the transfer function is infinite.2. The transfer function can be specified by the values of zeros, poles, and gain.

    The poles and zeros of a transfer function are complex numbers, so can be represented by points on the complex plane,which results in a pole-zero plot. In a pole-zero plot, a pole is marked with a cross and a zero is marked with a circle.

    7.2 Stability

    A system is called to be stable if the output dies away as t ; unstable if the output tends to infinity ast ; criticallystable if the output does not die away or increase to infinity but tends to some finite but non-zero values when subject to animpulse input.

    ExampleCheck the stability of the systems described by

    G1(s) = 1

    s + 2 G2(s) =

    1

    s 2Solution: Let input be a unit impulse function, that is, U(s) = 1. Then the output in s-domain is

    Y1(s) =G1(s)U(s) = 1

    s+ 2, Y2(s) =G2(s)U(s) =

    1

    s 2Taking the inverse Laplace transform gives

    y1(t) = e2t 0, t

    y2(t) = e2t , t

    39

  • 8/13/2019 Control System basics

    44/69

    40 CHAPTER 7. THE ROUTH STABILITY CRITERION

    So, G1(s) is stable andG2(s) unstable.

    Stability Test:

    1. A system is stable if all the poles are in the open left-hand side of the s-plane, that is, all the poles have negative realparts.

    2. A system is unstable if one or more poles are in the open right-hand side of the s-plane, that is, one or more poles have

    positive real parts.3. A system is critically stable if one or more poles are on the imaginary axis and the rest are in the open left-hand side

    of the s-plane, that is one or more poles have zero parts and the rest have negative real parts.

    Example 1Consider the following systems

    (a) s 1s2 4s + 4 (b)

    2(s + 1)

    (s+ 1)(s + 2)(s 3) (c) (s+ 3)(s 1)

    s(s+ 2)(s + 3)(s+ 4) (d)

    s+ 4

    s2 + s + 3 (e)

    1

    s2 + s + 1

    Find poles and zeros for the systems, plot them in s-plane, and check the stability.

    Solution

    (a) s1s24s+4 =

    s1(s2)2 . p1 = 2, p2 = 2, z1 = 1. It is unstable.

    (b) p1= 1, p2 = 2, p3 = 3, z1 = 1. It is unstable.(c) p1= 0, p2 = 2, p3 = 3, p4 = 4, z1 = 3, z2 = 1. It is stable.(d) s+4

    s2+s+3 = s+4

    [s( 12+j1

    2

    11)][s( 12j

    1

    2

    11)]

    . p1 = 12 +j 12

    11, p2 = 12j 12

    11, z1 = 4. It is stable.

    (e) 1s2+s+1 =

    1

    [s( 12+j 123)][s( 12j 12

    3)]

    . p1 = 12 + j 12

    3, p2= 12j 12

    3. It is stable.

    Example 2Find transfer functions and check stabilities for the systems

    (a) poles: -1, -2; zeros: no; gain: 2;

    (b) poles: 1, -2; zeros: 0; gain: 10;

    (c) poles: -2+j1, -2-j1; zeros: 1; gain: -5;

    (d) poles: 1+j2, 1-j2; zeros: -1; gain: 7.

    Solution:(a) 2(s+1)(s+2) is stable.

    (b) 10s(s1)(s+2) is unstable.

    (c) 5(s1)

    [s(2+j)][s(2j)] = 5(s1)(s+2)2+1 =

    5(s1)s2+4s+5 is stable.

    (d) 7(s+1)[s(1+j2)][s(1j2)] = 7(s+1)(s1)2+4 =

    7(s+1)s22s+5 is unstable.

    7.3 The Routh Stability Criterion

    We have discussed that the stability of a system can be determined by checking the positions of poles in the s-plane, which

    means that we need to find roots of the denominator of a transfer function. In general, it is not a easy thing to find roots ofthe polynomialans

    n + an1sn1 + an2s

    n2 + + a1s + a0with n >3.

    The Routh criterion can be used to determine the stability of a high order system without finding roots of its denominator.The procedure is given below.

    1. Write the characteristic equation in the form of

    ansn + an1s

    n1 + an2sn2 + + a1s + a0

    with an > 0

  • 8/13/2019 Control System basics

    45/69

    7.3. THE ROUTH STABILITY CRITERION 41

    2. If any of the coefficients are zero or negative, then there is a root or roots that are imaginary or have positive realparts, that is, the system is either critically stable or unstable.

    3. If all coefficients are positive, arrange the coefficients of the polynomial according to following pattern.

    sn an an2 an4 sn1 an1 an3 an5 sn2 b1 b2 b3

    sn3 c1 c2 c3 ...

    ......

    ...s2 x1 x2 x3s1 y1 y2s0 z1

    where

    b1 = an2

    anan1

    an3

    b2 = an4

    anan1

    an5

    c1 = an3

    an1b1

    b2

    c2 = an5

    an1b1

    b3

    Routh stability criterion states that the number of roots of the characteristic equation with positive real parts is equalto the number of changes in sign of the coefficients of the first column of the Routh array. If all elements in the firstcolumn are positive, then the system is stable. If there are negative elements in the first column, then the system isunstable.

    Example 1Check the stability ofG(s) = 2s+1s4+2s3+3s2+4s+1 .

    Solution:

    1. All coefficients of the characteristic equation are positive.

    2. Routh array is given below.s4 1 3 1s3 2 4s2 1

    = 3 12 4

    1

    = 1 12 0

    s1 2

    = 4 21 1

    s0 1

    = 1 12 0

    All the elements in the first column are positive, so the system is stable.

    Example 2Check the stability ofG(s) = 2s+1s4+s3+s2+4s+1 .

    Solution:

    1. All coefficients of the characteristic equation are positive.

    2. Routh array is given below.s4 1 1 1s3 1 4s2 3 = 1 11 4 1 = 1 11 0s1 13

    3

    = 4 13 1

    s0 1

    = 1 313

    3

    0

    The first column has a negative element, so the system is unstable.

  • 8/13/2019 Control System basics

    46/69

    42 CHAPTER 7. THE ROUTH STABILITY CRITERION

    Example 3The systemG(s) = 2s+1s54s4+3s3+2s2+5s+2 is unstable because there is a negative coefficient in the denominator.

    Example 4Find k so that the system with G1(s) = 10s+1 andG1(s) =

    1s(s+4) , as shown in the figure below is stable.

    Solution:

    1. The closed-loop transfer function is computed below.

    G(s) = G(s)

    1 + G(s)H(s)=

    10s+1

    1s(s+4)

    1 + 10s+1

    1s(s+4)

    k

    = 10

    (s+ 1)s(s + 4) + 10k =

    10

    s(s2 + 5s + 4) + 10k

    = 10

    s3

    + 5s2

    + 4s + 10k

    2. It follows from Routh-Hurwitz criterion that 10k >0, that is k >0.

    3. Routh array is given below.s3 1 4s2 5 10ks1 4 2k = 4 15 10ks0 10k

    = 10k 542k 0

    It follows from Routh-Hurwitz criterion that 4 2k >0, that is, k 0, that is k >0. So 0 < k

  • 8/13/2019 Control System basics

    47/69

    Chapter 8

    Root Locus Analysis

    8.1 Definition

    Consider a closed-loop system described by the following block diagram.

    The open-loop transfer function for the system is Go(s) = ks+1 with pole s = 1. The closed-loop transfer function is

    G(s) = Go(s)

    1 + Go(s)=

    ks+1

    1 + ks+1

    = k

    s+ 1 + k

    with pole(1 + k).It is evident that the closed-loop pole changes as k changes. For instance, the closed-loop pole is -1, which is the same as

    the open-loop pole, when k = 0, -2 for k = 1, -3 fork = 2, and so on. A curve of the closed-loop pole versusk for k >0 canbe plotted in s-plane. Such a curve is called the root locus, which is shown below.

    Now consider a second-order system with the open-loop transfer function Go(s) = k

    s2+4s+1 with open-loop poles 2 +

    3

    and2 3. Its closed-loop transfer function is computed below.

    G(s) = Go(s)

    1 + Go(s)=

    ks2+4s+1

    1 + ks2+4s+1

    = k

    s2 + 4s + 1 + k

    43

  • 8/13/2019 Control System basics

    48/69

    44 CHAPTER 8. ROOT LOCUS ANALYSIS

    The closed-loop poles, that is, roots of the characteristic equation s2 + 4s + 1 + k= 0, are

    p=4

    16 4(1+ k)

    2 = 2

    4 (1 + k) = 2

    3 k

    As k = 0, the closed-loop poles are2 + 3 and2 3, which are the open-loop poles. The system is overdamped.As 0< k 3, the closed-loop poles are2 +jk 3 and2 jk 3. The system is underdamped.The root locus plot for this system looks like the graph shown above.

    From these two example, we can make the following observations.

    1. The root locus is a locus of the closed-loop poles in s-plane as the gain increases from 0.

    2. The root locus starts at the open-loop poles.

    8.2 Construction

    Now consider a closed-loop system with unity feedback. Assume that the open-loop transfer function is given by

    Go(s) =k(s z1)(s z2) (s zm)

    (s p1)(s p2) (s pn)with open-loop gaink, open-loop zerosz1, z2, , zm, and open-loop poles p1, p2, , pn. Its closed-loop transfer function isgiven by

    G(s) = Go(s)

    1 +Go(s)

    The closed-loop characteristic equation is

    1 + Go(s) = 1 +k(s z1)(s z2) (s zm)

    (s p1)(s p2) (s pn) = 0

    that is,k(s z1)(s z2) (s zm)(s p1)(s p2) (s pn) = 1 = 1(odd multiple of)

    which implies that the phase condition

    [(s z1) + (s z2) + + (s zm)] [(s p1) + (s p2) + + (s pn)] = odd multiple of

    and magnitude condition| (s z1) || (s z2) | | (s zm) || (s p1) || (s p2) | | (s pn) | = 1

    must be satisfied.

    Construction of Root Loci

    1. The number of root loci is equal to the degree n of the open-loop characteristic equation or the order of the closed-looptransfer function.

    2. The root loci start at the open-loop poles and end at the open-loop zeros. If there are more poles than zeros, then mloci end at the open-loop zeros and the remaining (n-m) loci end at infinity.

    3. The root loci are symmetrical with respect to the real axis.

    4. Portions of the real axis are sections of root loci if the number of poles and zeros lying on the axis to the right of theportion is odd.

  • 8/13/2019 Control System basics

    49/69

    8.2. CONSTRUCTION 45

    5. Those loci terminating at infinity tend towards asymptotes at angles to the positive real axis of

    n m , 3

    n m , 5

    n m , ,[2(n m) 1]

    n m6. The asymptotes intersect the real axis at a point, called the center of gravity or centroid of the asymptotes, given by

    (p1+p2+ +pn) (z1+ z2+ + zn)n m

    7. The intersection of root loci with the imaginary axis can be found by calculating those values ofk which result in theexistence of imaginary roots, that is, solutions to the closed-loop characteristic equation with s =j.

    8. The term breakaway point is used for the case that two or more loci meet at a point and then subsequently breakaway from that point along separate paths. The breakaway point can be found by solving the closed-loop characteristicequation for k and solving

    dk

    ds = 0

    for s.

    9. The angle of departure of a locus at k = 0 from a complex pole and the angle of arrival of a locus at k = at acomplex zero can be determined by using the phase condition.

    Example 1Draw root loci for the unity feedback system with open-loop transfer function

    Go(s) = k(s+ 1)

    (s+ 2 +j3)(s + 2 j3)

    Solution:

    1. Find the closed-loop characteristic equation:

    G(s) = Go(s)

    1 + Go(s)=

    k(s+1)(s+2+j3)(s+2j3)

    1 + k(s+1)

    (s+2+j3)(s+2j3)=

    k(s + 1)

    (s+ 2 +j3)(s + 2 j3) + k(s + 1)

    which means the closed-loop characteristic equation is

    (s + 2 +j3)(s+ 2 j3) + k(s + 1) = 0

    2. Since Go(s) is of second order, there are two loci.

    3. Open-loop poles are2 +j3 and2 j3 and zeros are -1.4. The portion of the root loci on the real axis is from -1 tobecause to its right there is only one zero.5. The angle of the asymptote is

    nm = 21 = = 180

    , which implies that the asymptote is the real axis.

    6. Since the asymptote is the real axis, the intersection with the real axis has no significance.

    7. The intersection of loci with the imaginary axis can be determined by solving the characteristic equation withs = jfor k and , that is

    (j + 2 +j3)(j + 2 j3) + k(j + 1) = 0(j + 2)2 (j3)2 +jk+ k = 0

    (j)2 + 4j + 4 + 9 + jk+ k =

    2 +j(4 + k)+ 13 + k = 0which means that

    (4 + k) = 0

    2 + 13 + k = 0

  • 8/13/2019 Control System basics

    50/69

    46 CHAPTER 8. ROOT LOCUS ANALYSIS

    Solving these equations for k and gives k = 4 and = 13 + k = 13 4 = 3. In this course, the root lociare restricted to the case with positive k . So a negativek means there are no intersections between the root loci andthe imaginary axis, that is, the root loci does not cross the imaginary axis.

    8. The arrival point is determined by solving

    (s + 2 +j3)(s+ 2 j3) + k(s + 1) = 0

    for k , which is

    k= (s + 2 +j3)(s + 2 j3)s + 1

    = s2 + 4s + 13

    s + 1

    and letting dkds

    = 0, that is,

    0 =dk

    ds = (2s + 4)(s + 1) (s

    2 + 4s + 13)

    (s+ 1)2

    This is equivalent to

    (2s + 4)(s + 1)

    (s2 + 4s + 13) = 2s2 + 6s + 4

    s2

    4s

    13 = s2 + 2s

    9 = 0

    Solutions to this equation are

    s=2 4 + 36

    2 = 1

    10 = 4.1623, 2.1623

    Onlys = 4.1623 is on the root loci.

    9. There is one pair of complex poles and no complex zeros. The angle of departure of the locus at k = 0 from the complexpoles = 2 +j3 is determined by using the phase condition

    (s z1) [(s p1) + (s p2)] = odd multiple of 180

  • 8/13/2019 Control System basics

    51/69

    8.2. CONSTRUCTION 47

    with s being a point on the root locus very close to s = 2 +j3.

    (s z1) = 180 tan1

    3

    1

    = 108.4

    (s p2) = 90

    Thus,

    108.4 90 (s p1) = 180Solving this equation gives (s p1) =180 108.4+ 90 = 198.4, 161.6. These two angles are actually thesame because 198.4+ 161.6= 360.

    Example 2 Given a closed-loop system with unity feedback, which has an open-loop transfer function with poles -1,-2, -3, no zero and gain k. Draw root loci for this system and find the ranges ofk for the closed-loop system to be stable,critically stable and unstable.

    Solution:

    1. Find the open-loop transfer function

    Go(s) = k

    (s+ 1)(s+ 2)(s + 3)2. Find the closed-loop characteristic equation:

    G(s) = Go(s)

    1 + Go(s) =

    k(s+1)(s+2)(s+3)

    1 + k(s+1)(s+2)(s+3)=

    k

    (s + 1)(s+ 2)(s + 3) +k

    which means the closed-loop characteristic equation is

    (s+ 1)(s+ 2)(s + 3) + k= 0

    3. Since Go(s) is of third order, there are three loci.

    4. Open-loop poles are1,2,3 and zeros are none.5. The portion of the root loci on the real axis is from -1 to2 because to its right there is one pole and from3 to

    because to its right there is three poles.

    6. The angles of the asymptotes are nm =

    30 =

    3 and

    3nm =

    30 =.

    7. The intersection with the real axis isp1+p2+p3

    n m =1 2 3

    3 0 = 2

    8. The intersection of loci with the imaginary axis can be determined by solving the characteristic equation withs = jfor k and , that is

    (j + 1)(j + 2)(j + 3) + k = 0(2 +j3+ 2)(j + 3) + k = 0

    j3 32 +j2 32 +j9+ 6 + k = 06 + k 62 +j(11 3) = 0

    which means that

    6 + k 62 = 011 3 = 0

    Solving these equations for k and gives = 11 and k = 62 6 = 66 6 = 60.

  • 8/13/2019 Control System basics

    52/69

    48 CHAPTER 8. ROOT LOCUS ANALYSIS

    9. The breakaway point is determined by solving

    (s+ 1)(s+ 2)(s + 3) + k= 0

    for k , which isk= (s + 1)(s+ 2)(s + 3)

    and letting dkds

    = 0, that is,

    0 =dk

    ds = [(s +2)(s +3)+(s +1)(s +3)+(s +1)(s +2)] = [s2 + 5s + 6 + s2 + 4s + 3 + s2 + 3s +2] = [3s2 + 12s +11]

    This is equivalent to3s2 + 12s + 11 = 0

    Solutions to this equation are

    s=12 144 132

    6 = 1.42, 2.58

    Onlys = 1.42 is on the root loci.

    10. There is no complex poles and zeros, so there is no need to determine the departure or arrival angles.

    11. Whenk = 60, the system is critically stable because the closed-loop poles are on the imaginary axis. When 0< k 60, the systemis unstable because two closed-loop poles are in the open right-hand side of s-plane.

  • 8/13/2019 Control System basics

    53/69

    Chapter 9

    PID Controllers

    Proportional Controller: In t-domain, y(t) =KPe(t); in s-domainY(s) =KpE(s)Integral Controller: In t-domain, y (t) = KI

    e(t)dt; in s-domain Y(s) = KI

    s E(s)

    Derivative Controller: In t-domain, y(t) =KDde(t)dt

    ; in s-domain Y(s) =KDsE(s)

    PID Controller: In t-domain, y (t) = KPe(t) + KI e(t)dt+ KD de(t)dt ;. In s-domainY(s) =KpE(s) + KIs E(s) + KDsE(s) = (Kp+

    KIs + KDs)E(s).

    The block diagrams for these controllers are shown below.

    49

  • 8/13/2019 Control System basics

    54/69

    50 CHAPTER 9. PID CONTROLLERS

    These controllers can be implemented by using op-amps, as shown below.

    ExampleConsider a system with a transfer function ofGp(s) = 1s(s+1) . Answer the following questions.

    1. Check the stability of the system.

    2. Calculate the steady-state error with a unit step input and unit ramp input.

    3. Design a proportional controller to stabilize the system and achieve the steady-state error with a ramp input of 10%and find 5% settling time.

    4. Can you find an integral controller to stabilize the system?

    5. Design a PI controller to stabilize the system and to reduce the steady-state error with a ramp input to 0.

    6. Design a PID controller withKI= 0.5KP andKD = 0.5KPso that the overshoot is less than 25%.

    Solution:

    1. The system is critically stable.

    2. The error in s-domain is

    E(s) = U(s) Y(s) =U(s) 1s(s+ 1)

    U(s) =

    1 1

    s(s+ 1)

    U(s) =

    s2 + s 1s(s + 1)

    U(s)

    The steady-state error is

    ess= lims0

    sE(s) =

    lims0s

    s2+s1s(s+1)

    1s

    = U(s) = 1s

    lims0ss2+s1s(s+1)

    1s2

    = U(s) = 1s2

  • 8/13/2019 Control System basics

    55/69

    51

    3. The closed-loop system with a proportional controller is shown below.

    It is clearly seen that the open-loop transfer function is Go(s) = KPGp(s) = KPs(s+1) and the error in s-domain is

    E(s) = 11+Go(s)U(s). So with U(s) = 1s2

    andess= 0.1, we have

    0.1 lims0

    sE(s) = lims0

    s

    1 + Go(s)

    1

    s2= lim

    s01

    s + sGo(s)=

    1

    lims0sGo(s)=

    1

    lims0KPs+1

    = 1

    KP

    which implies that KP 10. So, chooseKP= 10.The closed-loop transfer function is

    Gc(s) = Go(s)

    1 + Go(s) =

    KPs(s+1)

    1 + KPs(s+1)

    = KP

    s2 + s + KP

    Comparing this with the standard form for a second-order system gives

    2n = KP n =

    KP =

    102n = 1 = 12n = 1210

    The 5% settling time is

    ts= 3

    n=

    3

    2

    10 10 = 0.15s

    4. The closed-loop system with an integral controller is shown below.

    The closed-loop transfer function is

    Gc(s) =

    KIs

    1s(s+1)

    1 + KIs

    1s(s+1)

    = KI

    s2(s + 1) + KI=

    KIs3 + s2 + KI

    Routh array is given as follows:s3 1 0s2 1 KIs1

    K

    I= 0 11KIs0 KI=KI 1KI 0

    The system is stable only ifKI>0 andKI>0. It is impossible. So, there is no solution to the problem.

    5. The closed-loop system with a PI controller is shown below.

  • 8/13/2019 Control System basics

    56/69

    52 CHAPTER 9. PID CONTROLLERS

    The open-loop transfer function is

    Go(s) =

    KP+

    KIs

    1

    s(s + 1)=

    KPs + KIs2(s+ 1)

    The error in s-domain is

    E(s) = 1

    1 + Go(s)U(s)

    The steady-state error is

    ess = lims0

    sE(s)

    =

    lims0s

    11+Go(s)

    1s

    = U(s) = 1s

    lims0s 1

    1+Go(s)1s2

    = U(s) = 1s2

    =

    1

    1+lims0Go(s)= 0 U(s) = 1

    s1

    lims0sGo(s) = 0 U(s) = 1

    s2

    So the PI controller with any KP andKIcan reduce errors to zero for both step and ramp input.

    The closed-loop transfer function is

    Gc(s) = Go(s)

    1 + Go(s)=

    KPs+KIs2(s+1)

    1 + KPs+KIs2(s+1)

    = KPs + KI

    s3 + s2 + KPs+ KI

    Routh array is given as follows:

    s3 1 KPs2 1 KIs1 KP 11KI=KP KIs0 KI 1KPKI

    KP

    0 = KI

    The system is stable only ifKP >, KP KI>0 and KI>0, that is KP > KI andKI>0.

    6. The closed-loop system with a PID controller is shown below.

    The closed-loop transfer function is

    Gc(s) =

    KP+

    0.5KPs

    + 0.5KPs

    1s(s+1)

    1 + KP+ 0.5KP

    s + 0.5KPs

    1s(s+1)

    = 0.5KPs2 + KPs+ 0.5KP

    s2(s+ 1) + 0.5KPs2 + KPs+ 0.5KP

    = 0.5KP(s + 1)2

    s2(s+ 1) + 0.5KP(s + 1)2

    = 0.5KP(s+ 1)

    s2 + 0.5KPs+ 0.5KP

    Comparing this with the standard form for a second-order system gives

    2n= 0.5KP n =

    0.5KP2n = 0.5KP =2n 2= n

  • 8/13/2019 Control System basics

    57/69

    53

    It follows from the requirement on overshoot that

    e

    12 0.25

    1 2 ln 0.25

    1

    2

    ln 0.25 = ln

    1

    0.25

    = ln 4

    ln 4

    1 222 (ln4)2 (1 2) = (ln4)2 (ln4)2 2

    2 + (ln 4)2

    2 (ln4)2

    2 (ln4)2

    2 + (ln 4)2

    ln 42 + (ln 4)

    2= 0.4037

    Choose = 0.5. Thenn = 2= 1, so 0.5KP =2n = 1, that is, KP = 2.

  • 8/13/2019 Control System basics

    58/69

    54 CHAPTER 9. PID CONTROLLERS

  • 8/13/2019 Control System basics

    59/69

    Chapter 10

    Frequency Response

    10.1 Steady-State Response

    Frequency response is the output of a system when subject to a sinusoidal input. The steady-state response of a system toa sinusoidal input u(t) =Usin(t) is also a sinusoidal function

    yss(t) =Y sin(t + )

    with Y =U| G(j)|and = [G(j )]. Here G(j ) is called frequency response function. This will be illustrated by thefollowing example.

    Example 1Find the output of a system G(s) = 1s+2 with a sinusoidal input u(t) =a sin(t).

    Solution: The Laplace transform of the input isU(s) = as2+2 and the output in s-domain is

    Y(s) =G(s)U(s) = 1

    s + 2

    a

    s2 + 2 =

    a

    (s + 2)(s2 + 2)

    Performing the partial fraction expansion gives

    Y(s) = A

    s+ 2+

    B s + C

    s2 + 2 =

    A(s2 + 2) + (Bs+ C)(s+ 2)

    (s + 2)(s2 + 2)

    As a result, we have

    a= A(s2 + 2) + (Bs+ C)(s+ 2)

    Let s = 2. Then, a = A(4 + 2), that is, A = a4+2 . Let s = 0. Then, a = A2 + 2C, that is,

    C=a A2

    2 =

    a a34+22

    = 4a

    2(4 + 2)=

    2a

    4 + 2

    Let s = 2. Then,a = A(4 + 2) + 4(2B+ C), that is, 0 = 4(2B+ C). As a result, we have

    B= C2

    = a4 + 2

    Therefore, the output is

    Y(s) =a

    4+2

    s + 2+

    a4+2 s + 2a4+2s2 + 2

    = a

    4 + 21

    s + 2 a

    4 + 2s

    s2 + 2+

    2a

    4 + 2

    s2 + 2

    Taking the inverse Laplace transform gives

    y(t) = a

    4 + 2e2t a

    4 + 2cos(t) +

    2a

    4 + 2sin(t)

    55

  • 8/13/2019 Control System basics

    60/69

    56 CHAPTER 10. FREQUENCY RESPONSE

    Now let cos = 24+2

    . Then, sin = 4+2

    and tan = sincos = 2 , that is, = tan12 . The output can be

    expressed as

    y(t) = a

    4 + 2e2t +

    a4 + 2

    [sin cos(t) + cos sin(t)] = a

    4 + 2e2t +

    a4 +2

    sin(t+ )

    When t goes to infinity, the first term will die away. Therefore, the steady-state response is given by

    yss(t) = a4 + 2

    sin(t+ )

    Now let us look at G(j) = 1j+2 . Its magnitude and angle are given by

    | G(j) | = 14 + 2

    [G(j)] = tan1

    2

    = tan1

    2

    So, the steady-state output can be written as

    yss(t) =a | G(j) | sin(t+ [G(j)])

    As a matter of fact, the steady-state response of any linear system G(s) to a sinusoidal input u(t) =a sin(t) is given by

    yss(t) =a | G(j) | sin(t+ [G(j)])

    ExampleWhat will be the steady-state response of a second order system

    d2y(t)

    dt2 + 3

    dy(t)

    dt + 10y(t) = 5u(t)

    when subject to the input u(t) = 2 sin(2t+ 70).

    Solution: By taking Laplace transform on both sides of the differential equation, we get

    s2

    Y(s) + 3sY(s) + 10Y(s) = 5U(s)

    Thus, the transfer function is

    G(s) = 5

    s2 + 3s + 10

    The frequency response function is

    G(j) = 5

    2 +j3+ 10 = 5

    10 2 +j3Its magnitude and phase angle at = 2 are given as follows:

    | G(j) | = 5

    [(10 22)2 + 9 22] =

    5

    6

    2

    [G(j)] = tan1 610 4

    = tan1(1) = 45

    Therefore, the solution to the problem is

    yss(t) = 5

    3

    2sin(2t + 25)

    10.2 Bode Plots

    Bode plot consists of two graphs, one of magnitude plotted against the frequency and one of the phase angle plotted againstthe frequency. The magnitude and phase angle are plotted using logarithmic scales.

  • 8/13/2019 Control System basics

    61/69

    10.2. BODE PLOTS 57

    It is common practice to express the magnitude in units of decibel (dB).

    Magnitude in dB = 20 lg | G(j) |Note that ifG(j) = G1(j)G2(j) Gn(j), then

    | G(j) |=| G1(j) |) | G2(j) | | Gn(j) |Taking logarithm to base 10 gives

    20lg | G(j) |= 20lg | G1(j) | +20 lg | G2(j) | + + 20 lg | Gn(j) |In addition, the phase angle satisfies

    [G(j)] = [G1(j)] + [G2(j)] + + [Gn(j)]Bode Plot for Constant Gain: G(s) = K, G(j) = K,| G(j)|= K, 20lg| G(j)|= 20lg K, and [G(j)] = 0.

    The Bode plot is given below.

    Bode Plot for

    1

    s : G(s) =

    1

    s , G(j) =

    1

    j ,| G(j) |= 1

    , 20lg | G(j) |= 20lg , and

    [G(j)] = 90.= 0.1rad/s 20lg | G(j) |= 20lg0.1 = 20dB= 1rad/s 20lg | G(j) |= 20lg1 = 0dB= 10rad/s 20lg | G(j) |= 20lg10 = 20dB

    The Bode plot is given below.

  • 8/13/2019 Control System basics

    62/69

    58 CHAPTER 10. FREQUENCY RESPONSE

    Bode Plot for s: G(s) =s, G(j) =j,| G(j) |=, 20lg | G(j) |= 20lg , and [G(j)] = 90.

    = 0.1rad/s 20lg | G(j) |= 20 lg0.1 = 20dB= 1rad/s 20lg | G(j) |= 20 lg1 = 0dB= 10rad/s 20lg | G(j) |= 20lg 10 = 20dB

    The Bode plot is given below.

    Bode Plot for 1s+1 : G(s) =

    1s+1 , G(j) =

    11+j ,| G(j)|= 11+22 , 20lg| G(j )|=20lg

    1 + 22, and

    [G(j)] = tan1 .

    = 0.1 1

    rad/s 20lg | G(j) |= 20lg 1 + 0.01 20lg1 = 0dB[G(j)] = tan1 0.1 0

    = 1

    rad/s 20lg | G(j) |= 20lg 1 + 1 = 20 lg 2 = 3dB 0dB[G(j)] =

    tan1 1 =

    45

    = 10 1 rad/s 20lg | G(j) |= 20lg 1 + 10 20lg10 = 20dB[G(j)] = tan1 10 90

    The point = 1

    is called break-point. The Bode plot is given below.

    Bode Plot for s+ 1: G(s) = s+ 1, G(j ) = 1 +j ,| G(j)|= 1 + 22, 20lg|G(j )|= 20lg 1 + 22, and

  • 8/13/2019 Control System basics

    63/69

    10.2. BODE PLOTS 59

    [G(j)] = tan1 .

    = 0.1 1

    rad/s 20lg | G(j) |= 20lg 1 + 0.01 20lg1 = 0dB[G(j)] = tan1 0.1 0

    = 1

    rad/s 20lg | G(j) |= 20lg 1 + 1 = 20lg2 = 3dB 0dB[G(j)] = tan1 1 = 45

    = 10 1

    rad/s 20lg | G(j) |= 20lg 1 + 10 20lg 10 = 20dB

    [G(j)] = tan1

    10 90The point = 1

    is called break-point. The Bode plot is given below.

    ExampleDraw asymptotes of the Bode diagram for a system G(s) = 102s+1 .

    Solution: G(s) = 10 12s+1 =G1(s)G2(s). The Bode plot is shown below.

  • 8/13/2019 Control System basics

    64/69

    60 CHAPTER 10. FREQUENCY RESPONSE

  • 8/13/2019 Control System basics

    65/69

    Chapter 11

    Control of Discrete Processes and PLC

    11.1 Introduction

    As a simple illustration of what is meant by discrete process control, consider a dc motor starting process. Because of alarge starting current, we need to insert some resistors into the armature circuit of a dc motor. In order to keep the startingcurrent under a certain level, we need to design a circuit to implement the following operations:

    1. When the start button is pushed, the motors armature circuit is connected to its power supply and the machine startswith all resistorsR1, R2, andR3.

    2. R1 is cut out from the armature circuit 10s later.

    3. R2 is cut out 10s after R1 is cut out.

    61

  • 8/13/2019 Control System basics

    66/69

    62 CHAPTER 11. CONTROL OF DISCRETE PROCESSES AND PLC

    4. R3 is cut out 10s after R2 is cut out.

    5. The motor will stop whenever the stop button is pushed.

    This starting process can be implemented by a circuit composed of switches and relays.

    The normally open button BO will be closed when pushed, whereas the normally closed button B Cwill be open whenpushed.

    A relay contains a coil and some contacts, either normally open contacts or normally closed contacts, or both. Normallyopen contacts will be closed when the coil is energized and open when de-energized, whereas normally closed contacts willbe open when energized and closed when de-energized.

    Time-delay relays are a kind of relays. Normally open contacts will be closed after a certain delay when the coil isenergized and open immediately when de-energized, whereas normally closed contacts will be open after a certain delaywhen energized and closed immediately when de-energized.

    The starting process can also be implemented by using a Programmable Logic Controller (PLC).

    A PLC is a microprocessor-based controller that uses a programmable memory to store instructions and implementfunctions such as logic, sequencing, timing, counting, and arithmetic.

  • 8/13/2019 Control System basics

    67/69

    11.2. PLC PROGRAMMING 63

    11.2 PLC Programming

    A PLC can be programmed by using a simple form of language, ladder diagram. Writing a program is equivalent to drawinga switching circuit.

    A ladder diagram looks like a ladder. It consists of two vertical lines, representing the power rails, and several horizontallines, which are rungs of the ladder. In drawing a ladder diagram, the following conventions are adopted:

    1. Vertical lines represent power rails between which circuits are connected.

    2. Each rung (horizontal line) on the ladder defines one operation in the control process.

    3. Each rung must start with an input or inputs and must end with at least one output.

    4. A ladder diagram is read from top to bottom. The rung is read from left to right. When a PLC is in its run mode, itgoes through the entire ladder program to the end, and then promptly resumes at the start. This procedure of goingthrough all the rungs of the program is termed a cycle.

    11.3 Internal Relays

    In PLCs there are elements that are used to hold data and behave like relays, being able to be switched on or off or switchother devices on or off. These elements are termed as internal relays. Such internal relays do not exist as real-world switchingdevices, but are merely bits in the storage memory that behave the same way as an external relay.

    Consider the dc motor starting circuit, a latch circuit is needed to start and stop the motor. Such a latch circuit can beimplemented by an internal relay, as shown below.

  • 8/13/2019 Control System basics

    68/69

  • 8/13/2019 Control System basics

    69/69

    11.5. COUNTERS 65

    value, that is, events are added until the number reaches the preset value. When the counters reach the preset value, theircontacts change state. As an example, let us analyze the ladder diagram shown below.

    1. The counter is reset and ready for counting when there is a pulse to RST.

    2. Start counting pulses to CTD.3. When the counter reaches its preset value, stop counting and its normally open contact changes state becomes closed.

    The timing diagram for this ladder diagram is given below.